In an experiment, subjects were shown a series of images on a computer screen, appearing usually at the top but occas...

kens on June 10, 2020

December 2017 SEC 3 Q21

Can someone please explain why C is incorrect? Thanks in advance!

Reply
Create a free account to read and take part in forum discussions.

Already have an account? log in

shunhe on June 14, 2020

Hi @kenken,

Thanks for the question! So this one’s a classic. Let’s take a look at what the stimulus is saying first. An experiment’s undertaken where people are shown images on a screen that usually appear on top but sometimes on the bottom, and then they’re supposed to guess where the next image appears. They guessed correctly less than half the time, based on patterns they thought they saw. But if they guessed the top always, they would’ve been correct most of the time.
?Now we’re asked to find a statement that MUST also be true based on the statements above. Let’s take a look at (C), which tells us that there was NO predictable pattern that one could reasonably believe occurred in the series of images on the computer screen. Well, first of all, the “no” language should make you wary of this one, since that’s some pretty strong wording (compared to D, which is pretty weak wording, and so more likely to be right). Also, clearly, there was a predictable pattern that people believed occurred in the images, since that’s what the people were guessing based off of! And even though there were wrong, we can’t say for sure that there was NO predictable pattern. (C) is just too strongly worded, and so is incorrect.

Hope this helps! Feel free to ask any other questions that you might have.